What are the theorems in mathematics which can be proved using completely different ideas? ...

How does debian/ubuntu knows a package has a updated version

How come Sam didn't become Lord of Horn Hill?

Understanding Ceva's Theorem

Why is "Consequences inflicted." not a sentence?

How to tell that you are a giant?

List *all* the tuples!

Denied boarding although I have proper visa and documentation. To whom should I make a complaint?

Okay to merge included columns on otherwise identical indexes?

Why are there no cargo aircraft with "flying wing" design?

What is known about the Ubaid lizard-people figurines?

Why are Kinder Surprise Eggs illegal in the USA?

What is the logic behind the Maharil's explanation of why we don't say שעשה ניסים on Pesach?

Extract all GPU name, model and GPU ram

Short Story with Cinderella as a Voo-doo Witch

Why didn't this character "real die" when they blew their stack out in Altered Carbon?

Seeking colloquialism for “just because”

How to find out what spells would be useless to a blind NPC spellcaster?

How discoverable are IPv6 addresses and AAAA names by potential attackers?

String `!23` is replaced with `docker` in command line

When a candle burns, why does the top of wick glow if bottom of flame is hottest?

Why light coming from distant stars is not discrete?

What is Wonderstone and are there any references to it pre-1982?

Can a non-EU citizen traveling with me come with me through the EU passport line?

Error "illegal generic type for instanceof" when using local classes



What are the theorems in mathematics which can be proved using completely different ideas?



Announcing the arrival of Valued Associate #679: Cesar Manara
Planned maintenance scheduled April 17/18, 2019 at 00:00UTC (8:00pm US/Eastern)Proving $int_{0}^{infty} mathrm{e}^{-x^2} dx = frac{sqrt pi}{2}$What is the most unusual proof you know that $sqrt{2}$ is irrational?Proof that a Combination is an integerNeed to prove the sequence $a_n=1+frac{1}{2^2}+frac{1}{3^2}+cdots+frac{1}{n^2}$ convergesIs $(mathbb{Q},+)$ the direct product of two non-trivial subgroups?Question about Euler's approach to find $sum_{n=1}^inftyfrac1{n^2}=frac{pi^2}6$When are two proofs “the same”?Evaluatig: $int_{0}^{infty}{e^{ax^2}cos(bx)dx}$Does there exist a group that is both a free product and a direct product of nontrivial groups?What is the largest possible variance of a random variable on $[0; 1]$?What are some easily-stated recently proven theorems?Need a theoretical textbook for calculus, proof basedBook ref. request: “…starting from a mathematically amorphous problem and combining ideas from sources to produce new mathematics…”Russian Texts on GeometryThe Largest Gaps in the History of MathematicsWhat delimits the mathematical framework within which information compression limits (from entropy) are valid.Similar book to All the Mathematics You Missed but at a graduate level that is of a reasonable size?Theorems relating to the limitation of mathematicsVery Elementary books on Analytic Number TheorySolutions for “What is Mathematics?” by Courant












42












$begingroup$


I would like to know about theorems which can give different proofs using completely different techniques.




Motivation:

When I read from the book Proof from the Book, I saw there were many many proof for the same theorem using completely different fields of mathematics.

(There were Six proofs of the infinity of primes based on different ideas even using topology.)




I wonder How mathematicians give this kind of proofs.

If you know such theorems and proofs please share them with me. Thank you.










share|cite|improve this question











$endgroup$












  • $begingroup$
    when are two proofs the same
    $endgroup$
    – Bumblebee
    May 7 '15 at 6:38


















42












$begingroup$


I would like to know about theorems which can give different proofs using completely different techniques.




Motivation:

When I read from the book Proof from the Book, I saw there were many many proof for the same theorem using completely different fields of mathematics.

(There were Six proofs of the infinity of primes based on different ideas even using topology.)




I wonder How mathematicians give this kind of proofs.

If you know such theorems and proofs please share them with me. Thank you.










share|cite|improve this question











$endgroup$












  • $begingroup$
    when are two proofs the same
    $endgroup$
    – Bumblebee
    May 7 '15 at 6:38
















42












42








42


19



$begingroup$


I would like to know about theorems which can give different proofs using completely different techniques.




Motivation:

When I read from the book Proof from the Book, I saw there were many many proof for the same theorem using completely different fields of mathematics.

(There were Six proofs of the infinity of primes based on different ideas even using topology.)




I wonder How mathematicians give this kind of proofs.

If you know such theorems and proofs please share them with me. Thank you.










share|cite|improve this question











$endgroup$




I would like to know about theorems which can give different proofs using completely different techniques.




Motivation:

When I read from the book Proof from the Book, I saw there were many many proof for the same theorem using completely different fields of mathematics.

(There were Six proofs of the infinity of primes based on different ideas even using topology.)




I wonder How mathematicians give this kind of proofs.

If you know such theorems and proofs please share them with me. Thank you.







reference-request big-list alternative-proof






share|cite|improve this question















share|cite|improve this question













share|cite|improve this question




share|cite|improve this question








edited Aug 10 '15 at 10:49







Bumblebee

















asked Nov 7 '14 at 1:34









BumblebeeBumblebee

9,74912551




9,74912551












  • $begingroup$
    when are two proofs the same
    $endgroup$
    – Bumblebee
    May 7 '15 at 6:38




















  • $begingroup$
    when are two proofs the same
    $endgroup$
    – Bumblebee
    May 7 '15 at 6:38


















$begingroup$
when are two proofs the same
$endgroup$
– Bumblebee
May 7 '15 at 6:38






$begingroup$
when are two proofs the same
$endgroup$
– Bumblebee
May 7 '15 at 6:38












13 Answers
13






active

oldest

votes


















19












$begingroup$

One example of a theorem with multiple proofs is the Fundamental Theorem of Algebra (All polynomials in $mathbb{C}[x]$ have the "right number" of roots). One way to prove this is build up enough complex analysis to prove that every bounded entire function is constant. Another way is to build up algebraic topology and use facts about maps from balls and circles into the punctured plane. Both of these techniques are used specifically to show one such root exists (and once this is proved the rest of the proof is easy).



I think there are other possible proofs of the theorem but these are two I have seen.






share|cite|improve this answer









$endgroup$









  • 2




    $begingroup$
    See more proofs in the book The Fundamental Theorem of Algebra by Fine and Rosenberger.
    $endgroup$
    – lhf
    Jan 26 '15 at 10:33






  • 1




    $begingroup$
    You can prove the fundamental theorem of algebra using Lie theory: if $L / Bbb C$ were a finite extension, the complex projective space $PL$ would inherit the structure of a compact commutative Lie group from the multiplication on $L$. But compact commutative Lie groups are tori, and complex projective space in any positive dimension is not a torus. (Credit goes to Mariano Suárez-Alvarez; I learned this proof from a comment he posted somewhere.) You can also prove the FTA entirely algebraically, though I don't remember the details.
    $endgroup$
    – user98602
    May 6 '15 at 3:29








  • 3




    $begingroup$
    See mathoverflow.net/questions/10535/… where 44 answers have been posted.
    $endgroup$
    – Gerry Myerson
    Jul 15 '15 at 7:20






  • 1




    $begingroup$
    There is an algebraic proof that makes use of the Sylow theorems and basic Galois theory.
    $endgroup$
    – Slade
    Jul 15 '15 at 7:52



















15












$begingroup$

The Brouwer fixed-point theorem. The Wikipedia lists the following methods:




  • Homology

  • Stokes's theorem

  • Combinatorial (Sperner's lemma)

  • Reducing to the smooth case and using Sard's theorem

  • Reducing to the smooth case and using the COV theorem

  • Lefschetz fixed-point theorem

  • Using Hex






share|cite|improve this answer









$endgroup$













  • $begingroup$
    What is COV theorem?
    $endgroup$
    – Mizar
    Jan 28 '15 at 9:10










  • $begingroup$
    @Mizar, change of variables: en.wikipedia.org/wiki/….
    $endgroup$
    – Martín-Blas Pérez Pinilla
    Jan 28 '15 at 9:14












  • $begingroup$
    While we're at it, there are two proofs using the Brouwer degree for continuous maps of open sets in $mathbb{R}^n$. One is essentially the same as Hirsch's theorem based on the impossibility of a retraction onto $S^{n-1}$, and the other constructs a homotopy between the identity and identity minus the continuous map.
    $endgroup$
    – Gyu Eun Lee
    Mar 16 '15 at 7:14



















14












$begingroup$

A large rectangle is tiled with smaller rectangles. Each of the smaller rectangles has at least one integer side. Must the large rectangle have at least one integer side?



What if you replace 'integer' with 'rational' or 'algebraic'?



Here are fourteen proofs.






share|cite|improve this answer









$endgroup$













  • $begingroup$
    Thanks, this is very nice!
    $endgroup$
    – joriki
    Jul 27 '15 at 17:29



















14












$begingroup$

As a kind of extreme example, I've heard of this book, reputed to have given over 360 proofs of the Pythagorean theorem. I'm not sure how pairwise different they are, but there's bound to be a lot of variety.



Another one that sticks in my mind is the proof of the Cayley-Hamilton theorem
for real (or complex) matrices. There is an algebraic proof that works for any field at all, but for the reals and complexes, you can argue topologically that the matrix you are working with is the limit of a sequence of diagonalizable matrices, and since the approximations satisfy their characteristic polynomial, so does the limit.






share|cite|improve this answer











$endgroup$









  • 3




    $begingroup$
    +1, and interestingly, the geometric proof of Cayley-Hamilton even carries over to arbitrary fields when one uses the Zariski topology.
    $endgroup$
    – Hanno
    Jan 18 '15 at 13:15






  • 1




    $begingroup$
    I'm currently this this, which gives a number of different proofs of the Pythagorean Theorem. Some proofs are very similar to each other, but others are quite different.
    $endgroup$
    – Randy E
    Jan 18 '15 at 13:16





















11












$begingroup$


  1. $20$ different proofs 0f the Euler Formula
    $$color{Red}{V-E+F=2.}$$


  2. $10$ different proofs 0f the Gaussian Integral
    $$color{Green}{int_{mathbb{R}}e^{-x^2},mathrm{d}x=sqrtpi.}$$







share|cite|improve this answer











$endgroup$













  • $begingroup$
    This and this also provides some methods of evaluating Gaussian integral.
    $endgroup$
    – Bumblebee
    Jul 15 '15 at 8:17



















10












$begingroup$

A famous example is the law of quadratic reciprocity. Wikipedia says that
several hundred proofs of the law of quadratic reciprocity have been found.
Many details concerning proofs by different methods can be found at the question at MO.






share|cite|improve this answer











$endgroup$





















    6












    $begingroup$

    The first example that comes to my mind is the Cauchy-Schwarz inequality.



    Here there is a paper I found some time ago in which the authors claim (I write claim, because I did not check each proof) to be able to show twelve different proofs of the result.



    Actually, there is an entire book on inequalities that starts from this very basic one, with various proofs of it.






    share|cite|improve this answer









    $endgroup$





















      5












      $begingroup$

      There are several results of this kind: For example the period three implies chaos theorem (http://mathworld.wolfram.com/PeriodThreeTheorem.html) was proved by Li and York (in 1975) by using the intermediate values theorem for continuous functions in despite the result belongs to discrete dynamics. A generalization of this result (http://mathworld.wolfram.com/SharkovskysTheorem.html) was made before (in 1965) by A.N. Sharkovsky by using a very complicated and different method.



      Also, I note that the most know theorems of L. Riemann were proved by introducing new ideas based on complex analysis. Those of Euler are based on the series...etc.






      share|cite|improve this answer











      $endgroup$





















        4












        $begingroup$

        Irving Adler in 'a new look at geometry' gives a circular chain of proofs of alternate versions of the fifth postulate (that if a line striking each of two lines at the same angle, then the two lines do not cross at any distance).



        The idea of such a proof shows that all of these propositions are identical in function.






        share|cite|improve this answer









        $endgroup$





















          4












          $begingroup$

          Proofs that $sqrt{2}$
          is irrational.



          I know (at least) four,
          and there are lists of many more.
          I could go on,
          but would be a bore,
          so I'll let someone else
          have the floor.






          share|cite|improve this answer









          $endgroup$









          • 1




            $begingroup$
            Do a Google search for "proof square root of 2 is irrational". One link, which has a number of proofs, is here: en.wikipedia.org/wiki/Square_root_of_2
            $endgroup$
            – marty cohen
            May 22 '15 at 14:56










          • $begingroup$
            I found your this post. Thank you.
            $endgroup$
            – Bumblebee
            Jun 22 '15 at 7:43






          • 1




            $begingroup$
            Another link with may proofs is cut-the-knot.org/proofs/sq_root.shtml
            $endgroup$
            – Gerry Myerson
            Jul 21 '15 at 6:56






          • 1




            $begingroup$
            When writing your answer, did you mean to rhyme each time [: )]?
            $endgroup$
            – Antonio DJC
            Jun 11 '17 at 3:33










          • $begingroup$
            One thing is certain: my rhyming is rarely inadverten t.
            $endgroup$
            – marty cohen
            Jun 11 '17 at 4:49



















          4












          $begingroup$

          Robin Chapman gives 14 proofs of $$sum_{n=1}^{infty}{1over n^2}={pi^2over6}$$ here.






          share|cite|improve this answer









          $endgroup$













          • $begingroup$
            This article provides a beautiful non-proof (Euler's proof) of this identity. Thank you.
            $endgroup$
            – Bumblebee
            Jul 17 '15 at 8:50





















          3












          $begingroup$

          For $ngeq kinmathbb{N}$, you can prove that $dfrac{n!}{k!cdot(n-k)!}$ is an integer from a combinatorial/counting argument. Establish that the formula gives the number of ways to choose a subset of $k$ from a set of $n$, and that automatically makes it an integer.



          Or you can prove that $dfrac{n!}{k!cdot(n-k)!}$ is an integer by showing that the highest power of any prime $p$ dividing the denominator is less than the highest power of $p$ dividing the numerator.



          Several proofs here.






          share|cite|improve this answer











          $endgroup$









          • 3




            $begingroup$
            You can also prove this using group actions: $S_n$ acts on the set ${1, 2, dotsc, n}$, and the stabilizer of any $k$-set from that set is $S_ktimes S_{n-k}$. So by the orbit-stabilizer theorem, the number of such $k$-sets is the number of orbits, which is $frac{|S_n|}{|S_k||S_{n-k}|} = frac{n!}{k!(n-k)!}$.
            $endgroup$
            – rogerl
            Jul 27 '15 at 17:27



















          1












          $begingroup$

          Other examples of such many-proof theorems include:



          1) Harmonic series diverges



          There are at least 8 proofs of this fact based on different ideas.
          The first four of them can be found under this link: https://proofwiki.org/wiki/Harmonic_Series_is_Divergent



          The other four are of the following form:
          "If a subseries diverges, then the whole series diverges too. Now let's prove that the subseries of harmonic series, that formed only by reciprocals of primes diverges..."



          And there are at least four different ways to prove it: https://en.wikipedia.org/wiki/Divergence_of_the_sum_of_the_reciprocals_of_the_primes



          2) Variance of a random variable with support in $[0; 1]$ does not exceed $frac{1}{4}$



          There are at least three proofs of this, based on different ideas: What is the largest possible variance of a random variable on $[0; 1]$?



          3) No group is both a direct product and a direct product of non-trivial groups



          There are at least three proofs of this, based on different ideas: Does there exist a group that is both a free product and a direct product of nontrivial groups?



          4) $(mathbb{Q}, +)$ is not a direct product of any two non-trivial groups



          There are at least three proofs of this, based on different ideas: Is $(mathbb{Q},+)$ the direct product of two non-trivial subgroups?






          share|cite|improve this answer











          $endgroup$














            Your Answer








            StackExchange.ready(function() {
            var channelOptions = {
            tags: "".split(" "),
            id: "69"
            };
            initTagRenderer("".split(" "), "".split(" "), channelOptions);

            StackExchange.using("externalEditor", function() {
            // Have to fire editor after snippets, if snippets enabled
            if (StackExchange.settings.snippets.snippetsEnabled) {
            StackExchange.using("snippets", function() {
            createEditor();
            });
            }
            else {
            createEditor();
            }
            });

            function createEditor() {
            StackExchange.prepareEditor({
            heartbeatType: 'answer',
            autoActivateHeartbeat: false,
            convertImagesToLinks: true,
            noModals: true,
            showLowRepImageUploadWarning: true,
            reputationToPostImages: 10,
            bindNavPrevention: true,
            postfix: "",
            imageUploader: {
            brandingHtml: "Powered by u003ca class="icon-imgur-white" href="https://imgur.com/"u003eu003c/au003e",
            contentPolicyHtml: "User contributions licensed under u003ca href="https://creativecommons.org/licenses/by-sa/3.0/"u003ecc by-sa 3.0 with attribution requiredu003c/au003e u003ca href="https://stackoverflow.com/legal/content-policy"u003e(content policy)u003c/au003e",
            allowUrls: true
            },
            noCode: true, onDemand: true,
            discardSelector: ".discard-answer"
            ,immediatelyShowMarkdownHelp:true
            });


            }
            });














            draft saved

            draft discarded


















            StackExchange.ready(
            function () {
            StackExchange.openid.initPostLogin('.new-post-login', 'https%3a%2f%2fmath.stackexchange.com%2fquestions%2f1009922%2fwhat-are-the-theorems-in-mathematics-which-can-be-proved-using-completely-differ%23new-answer', 'question_page');
            }
            );

            Post as a guest















            Required, but never shown

























            13 Answers
            13






            active

            oldest

            votes








            13 Answers
            13






            active

            oldest

            votes









            active

            oldest

            votes






            active

            oldest

            votes









            19












            $begingroup$

            One example of a theorem with multiple proofs is the Fundamental Theorem of Algebra (All polynomials in $mathbb{C}[x]$ have the "right number" of roots). One way to prove this is build up enough complex analysis to prove that every bounded entire function is constant. Another way is to build up algebraic topology and use facts about maps from balls and circles into the punctured plane. Both of these techniques are used specifically to show one such root exists (and once this is proved the rest of the proof is easy).



            I think there are other possible proofs of the theorem but these are two I have seen.






            share|cite|improve this answer









            $endgroup$









            • 2




              $begingroup$
              See more proofs in the book The Fundamental Theorem of Algebra by Fine and Rosenberger.
              $endgroup$
              – lhf
              Jan 26 '15 at 10:33






            • 1




              $begingroup$
              You can prove the fundamental theorem of algebra using Lie theory: if $L / Bbb C$ were a finite extension, the complex projective space $PL$ would inherit the structure of a compact commutative Lie group from the multiplication on $L$. But compact commutative Lie groups are tori, and complex projective space in any positive dimension is not a torus. (Credit goes to Mariano Suárez-Alvarez; I learned this proof from a comment he posted somewhere.) You can also prove the FTA entirely algebraically, though I don't remember the details.
              $endgroup$
              – user98602
              May 6 '15 at 3:29








            • 3




              $begingroup$
              See mathoverflow.net/questions/10535/… where 44 answers have been posted.
              $endgroup$
              – Gerry Myerson
              Jul 15 '15 at 7:20






            • 1




              $begingroup$
              There is an algebraic proof that makes use of the Sylow theorems and basic Galois theory.
              $endgroup$
              – Slade
              Jul 15 '15 at 7:52
















            19












            $begingroup$

            One example of a theorem with multiple proofs is the Fundamental Theorem of Algebra (All polynomials in $mathbb{C}[x]$ have the "right number" of roots). One way to prove this is build up enough complex analysis to prove that every bounded entire function is constant. Another way is to build up algebraic topology and use facts about maps from balls and circles into the punctured plane. Both of these techniques are used specifically to show one such root exists (and once this is proved the rest of the proof is easy).



            I think there are other possible proofs of the theorem but these are two I have seen.






            share|cite|improve this answer









            $endgroup$









            • 2




              $begingroup$
              See more proofs in the book The Fundamental Theorem of Algebra by Fine and Rosenberger.
              $endgroup$
              – lhf
              Jan 26 '15 at 10:33






            • 1




              $begingroup$
              You can prove the fundamental theorem of algebra using Lie theory: if $L / Bbb C$ were a finite extension, the complex projective space $PL$ would inherit the structure of a compact commutative Lie group from the multiplication on $L$. But compact commutative Lie groups are tori, and complex projective space in any positive dimension is not a torus. (Credit goes to Mariano Suárez-Alvarez; I learned this proof from a comment he posted somewhere.) You can also prove the FTA entirely algebraically, though I don't remember the details.
              $endgroup$
              – user98602
              May 6 '15 at 3:29








            • 3




              $begingroup$
              See mathoverflow.net/questions/10535/… where 44 answers have been posted.
              $endgroup$
              – Gerry Myerson
              Jul 15 '15 at 7:20






            • 1




              $begingroup$
              There is an algebraic proof that makes use of the Sylow theorems and basic Galois theory.
              $endgroup$
              – Slade
              Jul 15 '15 at 7:52














            19












            19








            19





            $begingroup$

            One example of a theorem with multiple proofs is the Fundamental Theorem of Algebra (All polynomials in $mathbb{C}[x]$ have the "right number" of roots). One way to prove this is build up enough complex analysis to prove that every bounded entire function is constant. Another way is to build up algebraic topology and use facts about maps from balls and circles into the punctured plane. Both of these techniques are used specifically to show one such root exists (and once this is proved the rest of the proof is easy).



            I think there are other possible proofs of the theorem but these are two I have seen.






            share|cite|improve this answer









            $endgroup$



            One example of a theorem with multiple proofs is the Fundamental Theorem of Algebra (All polynomials in $mathbb{C}[x]$ have the "right number" of roots). One way to prove this is build up enough complex analysis to prove that every bounded entire function is constant. Another way is to build up algebraic topology and use facts about maps from balls and circles into the punctured plane. Both of these techniques are used specifically to show one such root exists (and once this is proved the rest of the proof is easy).



            I think there are other possible proofs of the theorem but these are two I have seen.







            share|cite|improve this answer












            share|cite|improve this answer



            share|cite|improve this answer










            answered Nov 8 '14 at 0:00







            user171177















            • 2




              $begingroup$
              See more proofs in the book The Fundamental Theorem of Algebra by Fine and Rosenberger.
              $endgroup$
              – lhf
              Jan 26 '15 at 10:33






            • 1




              $begingroup$
              You can prove the fundamental theorem of algebra using Lie theory: if $L / Bbb C$ were a finite extension, the complex projective space $PL$ would inherit the structure of a compact commutative Lie group from the multiplication on $L$. But compact commutative Lie groups are tori, and complex projective space in any positive dimension is not a torus. (Credit goes to Mariano Suárez-Alvarez; I learned this proof from a comment he posted somewhere.) You can also prove the FTA entirely algebraically, though I don't remember the details.
              $endgroup$
              – user98602
              May 6 '15 at 3:29








            • 3




              $begingroup$
              See mathoverflow.net/questions/10535/… where 44 answers have been posted.
              $endgroup$
              – Gerry Myerson
              Jul 15 '15 at 7:20






            • 1




              $begingroup$
              There is an algebraic proof that makes use of the Sylow theorems and basic Galois theory.
              $endgroup$
              – Slade
              Jul 15 '15 at 7:52














            • 2




              $begingroup$
              See more proofs in the book The Fundamental Theorem of Algebra by Fine and Rosenberger.
              $endgroup$
              – lhf
              Jan 26 '15 at 10:33






            • 1




              $begingroup$
              You can prove the fundamental theorem of algebra using Lie theory: if $L / Bbb C$ were a finite extension, the complex projective space $PL$ would inherit the structure of a compact commutative Lie group from the multiplication on $L$. But compact commutative Lie groups are tori, and complex projective space in any positive dimension is not a torus. (Credit goes to Mariano Suárez-Alvarez; I learned this proof from a comment he posted somewhere.) You can also prove the FTA entirely algebraically, though I don't remember the details.
              $endgroup$
              – user98602
              May 6 '15 at 3:29








            • 3




              $begingroup$
              See mathoverflow.net/questions/10535/… where 44 answers have been posted.
              $endgroup$
              – Gerry Myerson
              Jul 15 '15 at 7:20






            • 1




              $begingroup$
              There is an algebraic proof that makes use of the Sylow theorems and basic Galois theory.
              $endgroup$
              – Slade
              Jul 15 '15 at 7:52








            2




            2




            $begingroup$
            See more proofs in the book The Fundamental Theorem of Algebra by Fine and Rosenberger.
            $endgroup$
            – lhf
            Jan 26 '15 at 10:33




            $begingroup$
            See more proofs in the book The Fundamental Theorem of Algebra by Fine and Rosenberger.
            $endgroup$
            – lhf
            Jan 26 '15 at 10:33




            1




            1




            $begingroup$
            You can prove the fundamental theorem of algebra using Lie theory: if $L / Bbb C$ were a finite extension, the complex projective space $PL$ would inherit the structure of a compact commutative Lie group from the multiplication on $L$. But compact commutative Lie groups are tori, and complex projective space in any positive dimension is not a torus. (Credit goes to Mariano Suárez-Alvarez; I learned this proof from a comment he posted somewhere.) You can also prove the FTA entirely algebraically, though I don't remember the details.
            $endgroup$
            – user98602
            May 6 '15 at 3:29






            $begingroup$
            You can prove the fundamental theorem of algebra using Lie theory: if $L / Bbb C$ were a finite extension, the complex projective space $PL$ would inherit the structure of a compact commutative Lie group from the multiplication on $L$. But compact commutative Lie groups are tori, and complex projective space in any positive dimension is not a torus. (Credit goes to Mariano Suárez-Alvarez; I learned this proof from a comment he posted somewhere.) You can also prove the FTA entirely algebraically, though I don't remember the details.
            $endgroup$
            – user98602
            May 6 '15 at 3:29






            3




            3




            $begingroup$
            See mathoverflow.net/questions/10535/… where 44 answers have been posted.
            $endgroup$
            – Gerry Myerson
            Jul 15 '15 at 7:20




            $begingroup$
            See mathoverflow.net/questions/10535/… where 44 answers have been posted.
            $endgroup$
            – Gerry Myerson
            Jul 15 '15 at 7:20




            1




            1




            $begingroup$
            There is an algebraic proof that makes use of the Sylow theorems and basic Galois theory.
            $endgroup$
            – Slade
            Jul 15 '15 at 7:52




            $begingroup$
            There is an algebraic proof that makes use of the Sylow theorems and basic Galois theory.
            $endgroup$
            – Slade
            Jul 15 '15 at 7:52











            15












            $begingroup$

            The Brouwer fixed-point theorem. The Wikipedia lists the following methods:




            • Homology

            • Stokes's theorem

            • Combinatorial (Sperner's lemma)

            • Reducing to the smooth case and using Sard's theorem

            • Reducing to the smooth case and using the COV theorem

            • Lefschetz fixed-point theorem

            • Using Hex






            share|cite|improve this answer









            $endgroup$













            • $begingroup$
              What is COV theorem?
              $endgroup$
              – Mizar
              Jan 28 '15 at 9:10










            • $begingroup$
              @Mizar, change of variables: en.wikipedia.org/wiki/….
              $endgroup$
              – Martín-Blas Pérez Pinilla
              Jan 28 '15 at 9:14












            • $begingroup$
              While we're at it, there are two proofs using the Brouwer degree for continuous maps of open sets in $mathbb{R}^n$. One is essentially the same as Hirsch's theorem based on the impossibility of a retraction onto $S^{n-1}$, and the other constructs a homotopy between the identity and identity minus the continuous map.
              $endgroup$
              – Gyu Eun Lee
              Mar 16 '15 at 7:14
















            15












            $begingroup$

            The Brouwer fixed-point theorem. The Wikipedia lists the following methods:




            • Homology

            • Stokes's theorem

            • Combinatorial (Sperner's lemma)

            • Reducing to the smooth case and using Sard's theorem

            • Reducing to the smooth case and using the COV theorem

            • Lefschetz fixed-point theorem

            • Using Hex






            share|cite|improve this answer









            $endgroup$













            • $begingroup$
              What is COV theorem?
              $endgroup$
              – Mizar
              Jan 28 '15 at 9:10










            • $begingroup$
              @Mizar, change of variables: en.wikipedia.org/wiki/….
              $endgroup$
              – Martín-Blas Pérez Pinilla
              Jan 28 '15 at 9:14












            • $begingroup$
              While we're at it, there are two proofs using the Brouwer degree for continuous maps of open sets in $mathbb{R}^n$. One is essentially the same as Hirsch's theorem based on the impossibility of a retraction onto $S^{n-1}$, and the other constructs a homotopy between the identity and identity minus the continuous map.
              $endgroup$
              – Gyu Eun Lee
              Mar 16 '15 at 7:14














            15












            15








            15





            $begingroup$

            The Brouwer fixed-point theorem. The Wikipedia lists the following methods:




            • Homology

            • Stokes's theorem

            • Combinatorial (Sperner's lemma)

            • Reducing to the smooth case and using Sard's theorem

            • Reducing to the smooth case and using the COV theorem

            • Lefschetz fixed-point theorem

            • Using Hex






            share|cite|improve this answer









            $endgroup$



            The Brouwer fixed-point theorem. The Wikipedia lists the following methods:




            • Homology

            • Stokes's theorem

            • Combinatorial (Sperner's lemma)

            • Reducing to the smooth case and using Sard's theorem

            • Reducing to the smooth case and using the COV theorem

            • Lefschetz fixed-point theorem

            • Using Hex







            share|cite|improve this answer












            share|cite|improve this answer



            share|cite|improve this answer










            answered Jan 26 '15 at 10:15









            Martín-Blas Pérez PinillaMartín-Blas Pérez Pinilla

            35.5k42972




            35.5k42972












            • $begingroup$
              What is COV theorem?
              $endgroup$
              – Mizar
              Jan 28 '15 at 9:10










            • $begingroup$
              @Mizar, change of variables: en.wikipedia.org/wiki/….
              $endgroup$
              – Martín-Blas Pérez Pinilla
              Jan 28 '15 at 9:14












            • $begingroup$
              While we're at it, there are two proofs using the Brouwer degree for continuous maps of open sets in $mathbb{R}^n$. One is essentially the same as Hirsch's theorem based on the impossibility of a retraction onto $S^{n-1}$, and the other constructs a homotopy between the identity and identity minus the continuous map.
              $endgroup$
              – Gyu Eun Lee
              Mar 16 '15 at 7:14


















            • $begingroup$
              What is COV theorem?
              $endgroup$
              – Mizar
              Jan 28 '15 at 9:10










            • $begingroup$
              @Mizar, change of variables: en.wikipedia.org/wiki/….
              $endgroup$
              – Martín-Blas Pérez Pinilla
              Jan 28 '15 at 9:14












            • $begingroup$
              While we're at it, there are two proofs using the Brouwer degree for continuous maps of open sets in $mathbb{R}^n$. One is essentially the same as Hirsch's theorem based on the impossibility of a retraction onto $S^{n-1}$, and the other constructs a homotopy between the identity and identity minus the continuous map.
              $endgroup$
              – Gyu Eun Lee
              Mar 16 '15 at 7:14
















            $begingroup$
            What is COV theorem?
            $endgroup$
            – Mizar
            Jan 28 '15 at 9:10




            $begingroup$
            What is COV theorem?
            $endgroup$
            – Mizar
            Jan 28 '15 at 9:10












            $begingroup$
            @Mizar, change of variables: en.wikipedia.org/wiki/….
            $endgroup$
            – Martín-Blas Pérez Pinilla
            Jan 28 '15 at 9:14






            $begingroup$
            @Mizar, change of variables: en.wikipedia.org/wiki/….
            $endgroup$
            – Martín-Blas Pérez Pinilla
            Jan 28 '15 at 9:14














            $begingroup$
            While we're at it, there are two proofs using the Brouwer degree for continuous maps of open sets in $mathbb{R}^n$. One is essentially the same as Hirsch's theorem based on the impossibility of a retraction onto $S^{n-1}$, and the other constructs a homotopy between the identity and identity minus the continuous map.
            $endgroup$
            – Gyu Eun Lee
            Mar 16 '15 at 7:14




            $begingroup$
            While we're at it, there are two proofs using the Brouwer degree for continuous maps of open sets in $mathbb{R}^n$. One is essentially the same as Hirsch's theorem based on the impossibility of a retraction onto $S^{n-1}$, and the other constructs a homotopy between the identity and identity minus the continuous map.
            $endgroup$
            – Gyu Eun Lee
            Mar 16 '15 at 7:14











            14












            $begingroup$

            A large rectangle is tiled with smaller rectangles. Each of the smaller rectangles has at least one integer side. Must the large rectangle have at least one integer side?



            What if you replace 'integer' with 'rational' or 'algebraic'?



            Here are fourteen proofs.






            share|cite|improve this answer









            $endgroup$













            • $begingroup$
              Thanks, this is very nice!
              $endgroup$
              – joriki
              Jul 27 '15 at 17:29
















            14












            $begingroup$

            A large rectangle is tiled with smaller rectangles. Each of the smaller rectangles has at least one integer side. Must the large rectangle have at least one integer side?



            What if you replace 'integer' with 'rational' or 'algebraic'?



            Here are fourteen proofs.






            share|cite|improve this answer









            $endgroup$













            • $begingroup$
              Thanks, this is very nice!
              $endgroup$
              – joriki
              Jul 27 '15 at 17:29














            14












            14








            14





            $begingroup$

            A large rectangle is tiled with smaller rectangles. Each of the smaller rectangles has at least one integer side. Must the large rectangle have at least one integer side?



            What if you replace 'integer' with 'rational' or 'algebraic'?



            Here are fourteen proofs.






            share|cite|improve this answer









            $endgroup$



            A large rectangle is tiled with smaller rectangles. Each of the smaller rectangles has at least one integer side. Must the large rectangle have at least one integer side?



            What if you replace 'integer' with 'rational' or 'algebraic'?



            Here are fourteen proofs.







            share|cite|improve this answer












            share|cite|improve this answer



            share|cite|improve this answer










            answered Jan 18 '15 at 13:10









            LopsyLopsy

            3,7731423




            3,7731423












            • $begingroup$
              Thanks, this is very nice!
              $endgroup$
              – joriki
              Jul 27 '15 at 17:29


















            • $begingroup$
              Thanks, this is very nice!
              $endgroup$
              – joriki
              Jul 27 '15 at 17:29
















            $begingroup$
            Thanks, this is very nice!
            $endgroup$
            – joriki
            Jul 27 '15 at 17:29




            $begingroup$
            Thanks, this is very nice!
            $endgroup$
            – joriki
            Jul 27 '15 at 17:29











            14












            $begingroup$

            As a kind of extreme example, I've heard of this book, reputed to have given over 360 proofs of the Pythagorean theorem. I'm not sure how pairwise different they are, but there's bound to be a lot of variety.



            Another one that sticks in my mind is the proof of the Cayley-Hamilton theorem
            for real (or complex) matrices. There is an algebraic proof that works for any field at all, but for the reals and complexes, you can argue topologically that the matrix you are working with is the limit of a sequence of diagonalizable matrices, and since the approximations satisfy their characteristic polynomial, so does the limit.






            share|cite|improve this answer











            $endgroup$









            • 3




              $begingroup$
              +1, and interestingly, the geometric proof of Cayley-Hamilton even carries over to arbitrary fields when one uses the Zariski topology.
              $endgroup$
              – Hanno
              Jan 18 '15 at 13:15






            • 1




              $begingroup$
              I'm currently this this, which gives a number of different proofs of the Pythagorean Theorem. Some proofs are very similar to each other, but others are quite different.
              $endgroup$
              – Randy E
              Jan 18 '15 at 13:16


















            14












            $begingroup$

            As a kind of extreme example, I've heard of this book, reputed to have given over 360 proofs of the Pythagorean theorem. I'm not sure how pairwise different they are, but there's bound to be a lot of variety.



            Another one that sticks in my mind is the proof of the Cayley-Hamilton theorem
            for real (or complex) matrices. There is an algebraic proof that works for any field at all, but for the reals and complexes, you can argue topologically that the matrix you are working with is the limit of a sequence of diagonalizable matrices, and since the approximations satisfy their characteristic polynomial, so does the limit.






            share|cite|improve this answer











            $endgroup$









            • 3




              $begingroup$
              +1, and interestingly, the geometric proof of Cayley-Hamilton even carries over to arbitrary fields when one uses the Zariski topology.
              $endgroup$
              – Hanno
              Jan 18 '15 at 13:15






            • 1




              $begingroup$
              I'm currently this this, which gives a number of different proofs of the Pythagorean Theorem. Some proofs are very similar to each other, but others are quite different.
              $endgroup$
              – Randy E
              Jan 18 '15 at 13:16
















            14












            14








            14





            $begingroup$

            As a kind of extreme example, I've heard of this book, reputed to have given over 360 proofs of the Pythagorean theorem. I'm not sure how pairwise different they are, but there's bound to be a lot of variety.



            Another one that sticks in my mind is the proof of the Cayley-Hamilton theorem
            for real (or complex) matrices. There is an algebraic proof that works for any field at all, but for the reals and complexes, you can argue topologically that the matrix you are working with is the limit of a sequence of diagonalizable matrices, and since the approximations satisfy their characteristic polynomial, so does the limit.






            share|cite|improve this answer











            $endgroup$



            As a kind of extreme example, I've heard of this book, reputed to have given over 360 proofs of the Pythagorean theorem. I'm not sure how pairwise different they are, but there's bound to be a lot of variety.



            Another one that sticks in my mind is the proof of the Cayley-Hamilton theorem
            for real (or complex) matrices. There is an algebraic proof that works for any field at all, but for the reals and complexes, you can argue topologically that the matrix you are working with is the limit of a sequence of diagonalizable matrices, and since the approximations satisfy their characteristic polynomial, so does the limit.







            share|cite|improve this answer














            share|cite|improve this answer



            share|cite|improve this answer








            edited Jul 15 '15 at 6:58









            Bumblebee

            9,74912551




            9,74912551










            answered Jan 18 '15 at 12:59









            rschwiebrschwieb

            108k12104253




            108k12104253








            • 3




              $begingroup$
              +1, and interestingly, the geometric proof of Cayley-Hamilton even carries over to arbitrary fields when one uses the Zariski topology.
              $endgroup$
              – Hanno
              Jan 18 '15 at 13:15






            • 1




              $begingroup$
              I'm currently this this, which gives a number of different proofs of the Pythagorean Theorem. Some proofs are very similar to each other, but others are quite different.
              $endgroup$
              – Randy E
              Jan 18 '15 at 13:16
















            • 3




              $begingroup$
              +1, and interestingly, the geometric proof of Cayley-Hamilton even carries over to arbitrary fields when one uses the Zariski topology.
              $endgroup$
              – Hanno
              Jan 18 '15 at 13:15






            • 1




              $begingroup$
              I'm currently this this, which gives a number of different proofs of the Pythagorean Theorem. Some proofs are very similar to each other, but others are quite different.
              $endgroup$
              – Randy E
              Jan 18 '15 at 13:16










            3




            3




            $begingroup$
            +1, and interestingly, the geometric proof of Cayley-Hamilton even carries over to arbitrary fields when one uses the Zariski topology.
            $endgroup$
            – Hanno
            Jan 18 '15 at 13:15




            $begingroup$
            +1, and interestingly, the geometric proof of Cayley-Hamilton even carries over to arbitrary fields when one uses the Zariski topology.
            $endgroup$
            – Hanno
            Jan 18 '15 at 13:15




            1




            1




            $begingroup$
            I'm currently this this, which gives a number of different proofs of the Pythagorean Theorem. Some proofs are very similar to each other, but others are quite different.
            $endgroup$
            – Randy E
            Jan 18 '15 at 13:16






            $begingroup$
            I'm currently this this, which gives a number of different proofs of the Pythagorean Theorem. Some proofs are very similar to each other, but others are quite different.
            $endgroup$
            – Randy E
            Jan 18 '15 at 13:16













            11












            $begingroup$


            1. $20$ different proofs 0f the Euler Formula
              $$color{Red}{V-E+F=2.}$$


            2. $10$ different proofs 0f the Gaussian Integral
              $$color{Green}{int_{mathbb{R}}e^{-x^2},mathrm{d}x=sqrtpi.}$$







            share|cite|improve this answer











            $endgroup$













            • $begingroup$
              This and this also provides some methods of evaluating Gaussian integral.
              $endgroup$
              – Bumblebee
              Jul 15 '15 at 8:17
















            11












            $begingroup$


            1. $20$ different proofs 0f the Euler Formula
              $$color{Red}{V-E+F=2.}$$


            2. $10$ different proofs 0f the Gaussian Integral
              $$color{Green}{int_{mathbb{R}}e^{-x^2},mathrm{d}x=sqrtpi.}$$







            share|cite|improve this answer











            $endgroup$













            • $begingroup$
              This and this also provides some methods of evaluating Gaussian integral.
              $endgroup$
              – Bumblebee
              Jul 15 '15 at 8:17














            11












            11








            11





            $begingroup$


            1. $20$ different proofs 0f the Euler Formula
              $$color{Red}{V-E+F=2.}$$


            2. $10$ different proofs 0f the Gaussian Integral
              $$color{Green}{int_{mathbb{R}}e^{-x^2},mathrm{d}x=sqrtpi.}$$







            share|cite|improve this answer











            $endgroup$




            1. $20$ different proofs 0f the Euler Formula
              $$color{Red}{V-E+F=2.}$$


            2. $10$ different proofs 0f the Gaussian Integral
              $$color{Green}{int_{mathbb{R}}e^{-x^2},mathrm{d}x=sqrtpi.}$$








            share|cite|improve this answer














            share|cite|improve this answer



            share|cite|improve this answer








            edited Jan 30 '17 at 16:02

























            answered Jan 28 '15 at 8:56









            BumblebeeBumblebee

            9,74912551




            9,74912551












            • $begingroup$
              This and this also provides some methods of evaluating Gaussian integral.
              $endgroup$
              – Bumblebee
              Jul 15 '15 at 8:17


















            • $begingroup$
              This and this also provides some methods of evaluating Gaussian integral.
              $endgroup$
              – Bumblebee
              Jul 15 '15 at 8:17
















            $begingroup$
            This and this also provides some methods of evaluating Gaussian integral.
            $endgroup$
            – Bumblebee
            Jul 15 '15 at 8:17




            $begingroup$
            This and this also provides some methods of evaluating Gaussian integral.
            $endgroup$
            – Bumblebee
            Jul 15 '15 at 8:17











            10












            $begingroup$

            A famous example is the law of quadratic reciprocity. Wikipedia says that
            several hundred proofs of the law of quadratic reciprocity have been found.
            Many details concerning proofs by different methods can be found at the question at MO.






            share|cite|improve this answer











            $endgroup$


















              10












              $begingroup$

              A famous example is the law of quadratic reciprocity. Wikipedia says that
              several hundred proofs of the law of quadratic reciprocity have been found.
              Many details concerning proofs by different methods can be found at the question at MO.






              share|cite|improve this answer











              $endgroup$
















                10












                10








                10





                $begingroup$

                A famous example is the law of quadratic reciprocity. Wikipedia says that
                several hundred proofs of the law of quadratic reciprocity have been found.
                Many details concerning proofs by different methods can be found at the question at MO.






                share|cite|improve this answer











                $endgroup$



                A famous example is the law of quadratic reciprocity. Wikipedia says that
                several hundred proofs of the law of quadratic reciprocity have been found.
                Many details concerning proofs by different methods can be found at the question at MO.







                share|cite|improve this answer














                share|cite|improve this answer



                share|cite|improve this answer








                edited Apr 13 '17 at 12:58









                Community

                1




                1










                answered Apr 29 '15 at 8:06









                Dietrich BurdeDietrich Burde

                82.1k649107




                82.1k649107























                    6












                    $begingroup$

                    The first example that comes to my mind is the Cauchy-Schwarz inequality.



                    Here there is a paper I found some time ago in which the authors claim (I write claim, because I did not check each proof) to be able to show twelve different proofs of the result.



                    Actually, there is an entire book on inequalities that starts from this very basic one, with various proofs of it.






                    share|cite|improve this answer









                    $endgroup$


















                      6












                      $begingroup$

                      The first example that comes to my mind is the Cauchy-Schwarz inequality.



                      Here there is a paper I found some time ago in which the authors claim (I write claim, because I did not check each proof) to be able to show twelve different proofs of the result.



                      Actually, there is an entire book on inequalities that starts from this very basic one, with various proofs of it.






                      share|cite|improve this answer









                      $endgroup$
















                        6












                        6








                        6





                        $begingroup$

                        The first example that comes to my mind is the Cauchy-Schwarz inequality.



                        Here there is a paper I found some time ago in which the authors claim (I write claim, because I did not check each proof) to be able to show twelve different proofs of the result.



                        Actually, there is an entire book on inequalities that starts from this very basic one, with various proofs of it.






                        share|cite|improve this answer









                        $endgroup$



                        The first example that comes to my mind is the Cauchy-Schwarz inequality.



                        Here there is a paper I found some time ago in which the authors claim (I write claim, because I did not check each proof) to be able to show twelve different proofs of the result.



                        Actually, there is an entire book on inequalities that starts from this very basic one, with various proofs of it.







                        share|cite|improve this answer












                        share|cite|improve this answer



                        share|cite|improve this answer










                        answered Jan 28 '15 at 9:11









                        KolminKolmin

                        1,89411435




                        1,89411435























                            5












                            $begingroup$

                            There are several results of this kind: For example the period three implies chaos theorem (http://mathworld.wolfram.com/PeriodThreeTheorem.html) was proved by Li and York (in 1975) by using the intermediate values theorem for continuous functions in despite the result belongs to discrete dynamics. A generalization of this result (http://mathworld.wolfram.com/SharkovskysTheorem.html) was made before (in 1965) by A.N. Sharkovsky by using a very complicated and different method.



                            Also, I note that the most know theorems of L. Riemann were proved by introducing new ideas based on complex analysis. Those of Euler are based on the series...etc.






                            share|cite|improve this answer











                            $endgroup$


















                              5












                              $begingroup$

                              There are several results of this kind: For example the period three implies chaos theorem (http://mathworld.wolfram.com/PeriodThreeTheorem.html) was proved by Li and York (in 1975) by using the intermediate values theorem for continuous functions in despite the result belongs to discrete dynamics. A generalization of this result (http://mathworld.wolfram.com/SharkovskysTheorem.html) was made before (in 1965) by A.N. Sharkovsky by using a very complicated and different method.



                              Also, I note that the most know theorems of L. Riemann were proved by introducing new ideas based on complex analysis. Those of Euler are based on the series...etc.






                              share|cite|improve this answer











                              $endgroup$
















                                5












                                5








                                5





                                $begingroup$

                                There are several results of this kind: For example the period three implies chaos theorem (http://mathworld.wolfram.com/PeriodThreeTheorem.html) was proved by Li and York (in 1975) by using the intermediate values theorem for continuous functions in despite the result belongs to discrete dynamics. A generalization of this result (http://mathworld.wolfram.com/SharkovskysTheorem.html) was made before (in 1965) by A.N. Sharkovsky by using a very complicated and different method.



                                Also, I note that the most know theorems of L. Riemann were proved by introducing new ideas based on complex analysis. Those of Euler are based on the series...etc.






                                share|cite|improve this answer











                                $endgroup$



                                There are several results of this kind: For example the period three implies chaos theorem (http://mathworld.wolfram.com/PeriodThreeTheorem.html) was proved by Li and York (in 1975) by using the intermediate values theorem for continuous functions in despite the result belongs to discrete dynamics. A generalization of this result (http://mathworld.wolfram.com/SharkovskysTheorem.html) was made before (in 1965) by A.N. Sharkovsky by using a very complicated and different method.



                                Also, I note that the most know theorems of L. Riemann were proved by introducing new ideas based on complex analysis. Those of Euler are based on the series...etc.







                                share|cite|improve this answer














                                share|cite|improve this answer



                                share|cite|improve this answer








                                edited Jan 18 '15 at 13:04

























                                answered Jan 18 '15 at 12:48









                                DERDER

                                1,6631018




                                1,6631018























                                    4












                                    $begingroup$

                                    Irving Adler in 'a new look at geometry' gives a circular chain of proofs of alternate versions of the fifth postulate (that if a line striking each of two lines at the same angle, then the two lines do not cross at any distance).



                                    The idea of such a proof shows that all of these propositions are identical in function.






                                    share|cite|improve this answer









                                    $endgroup$


















                                      4












                                      $begingroup$

                                      Irving Adler in 'a new look at geometry' gives a circular chain of proofs of alternate versions of the fifth postulate (that if a line striking each of two lines at the same angle, then the two lines do not cross at any distance).



                                      The idea of such a proof shows that all of these propositions are identical in function.






                                      share|cite|improve this answer









                                      $endgroup$
















                                        4












                                        4








                                        4





                                        $begingroup$

                                        Irving Adler in 'a new look at geometry' gives a circular chain of proofs of alternate versions of the fifth postulate (that if a line striking each of two lines at the same angle, then the two lines do not cross at any distance).



                                        The idea of such a proof shows that all of these propositions are identical in function.






                                        share|cite|improve this answer









                                        $endgroup$



                                        Irving Adler in 'a new look at geometry' gives a circular chain of proofs of alternate versions of the fifth postulate (that if a line striking each of two lines at the same angle, then the two lines do not cross at any distance).



                                        The idea of such a proof shows that all of these propositions are identical in function.







                                        share|cite|improve this answer












                                        share|cite|improve this answer



                                        share|cite|improve this answer










                                        answered Jan 28 '15 at 9:33









                                        wendy.kriegerwendy.krieger

                                        5,86311427




                                        5,86311427























                                            4












                                            $begingroup$

                                            Proofs that $sqrt{2}$
                                            is irrational.



                                            I know (at least) four,
                                            and there are lists of many more.
                                            I could go on,
                                            but would be a bore,
                                            so I'll let someone else
                                            have the floor.






                                            share|cite|improve this answer









                                            $endgroup$









                                            • 1




                                              $begingroup$
                                              Do a Google search for "proof square root of 2 is irrational". One link, which has a number of proofs, is here: en.wikipedia.org/wiki/Square_root_of_2
                                              $endgroup$
                                              – marty cohen
                                              May 22 '15 at 14:56










                                            • $begingroup$
                                              I found your this post. Thank you.
                                              $endgroup$
                                              – Bumblebee
                                              Jun 22 '15 at 7:43






                                            • 1




                                              $begingroup$
                                              Another link with may proofs is cut-the-knot.org/proofs/sq_root.shtml
                                              $endgroup$
                                              – Gerry Myerson
                                              Jul 21 '15 at 6:56






                                            • 1




                                              $begingroup$
                                              When writing your answer, did you mean to rhyme each time [: )]?
                                              $endgroup$
                                              – Antonio DJC
                                              Jun 11 '17 at 3:33










                                            • $begingroup$
                                              One thing is certain: my rhyming is rarely inadverten t.
                                              $endgroup$
                                              – marty cohen
                                              Jun 11 '17 at 4:49
















                                            4












                                            $begingroup$

                                            Proofs that $sqrt{2}$
                                            is irrational.



                                            I know (at least) four,
                                            and there are lists of many more.
                                            I could go on,
                                            but would be a bore,
                                            so I'll let someone else
                                            have the floor.






                                            share|cite|improve this answer









                                            $endgroup$









                                            • 1




                                              $begingroup$
                                              Do a Google search for "proof square root of 2 is irrational". One link, which has a number of proofs, is here: en.wikipedia.org/wiki/Square_root_of_2
                                              $endgroup$
                                              – marty cohen
                                              May 22 '15 at 14:56










                                            • $begingroup$
                                              I found your this post. Thank you.
                                              $endgroup$
                                              – Bumblebee
                                              Jun 22 '15 at 7:43






                                            • 1




                                              $begingroup$
                                              Another link with may proofs is cut-the-knot.org/proofs/sq_root.shtml
                                              $endgroup$
                                              – Gerry Myerson
                                              Jul 21 '15 at 6:56






                                            • 1




                                              $begingroup$
                                              When writing your answer, did you mean to rhyme each time [: )]?
                                              $endgroup$
                                              – Antonio DJC
                                              Jun 11 '17 at 3:33










                                            • $begingroup$
                                              One thing is certain: my rhyming is rarely inadverten t.
                                              $endgroup$
                                              – marty cohen
                                              Jun 11 '17 at 4:49














                                            4












                                            4








                                            4





                                            $begingroup$

                                            Proofs that $sqrt{2}$
                                            is irrational.



                                            I know (at least) four,
                                            and there are lists of many more.
                                            I could go on,
                                            but would be a bore,
                                            so I'll let someone else
                                            have the floor.






                                            share|cite|improve this answer









                                            $endgroup$



                                            Proofs that $sqrt{2}$
                                            is irrational.



                                            I know (at least) four,
                                            and there are lists of many more.
                                            I could go on,
                                            but would be a bore,
                                            so I'll let someone else
                                            have the floor.







                                            share|cite|improve this answer












                                            share|cite|improve this answer



                                            share|cite|improve this answer










                                            answered May 6 '15 at 3:50









                                            marty cohenmarty cohen

                                            75.7k549130




                                            75.7k549130








                                            • 1




                                              $begingroup$
                                              Do a Google search for "proof square root of 2 is irrational". One link, which has a number of proofs, is here: en.wikipedia.org/wiki/Square_root_of_2
                                              $endgroup$
                                              – marty cohen
                                              May 22 '15 at 14:56










                                            • $begingroup$
                                              I found your this post. Thank you.
                                              $endgroup$
                                              – Bumblebee
                                              Jun 22 '15 at 7:43






                                            • 1




                                              $begingroup$
                                              Another link with may proofs is cut-the-knot.org/proofs/sq_root.shtml
                                              $endgroup$
                                              – Gerry Myerson
                                              Jul 21 '15 at 6:56






                                            • 1




                                              $begingroup$
                                              When writing your answer, did you mean to rhyme each time [: )]?
                                              $endgroup$
                                              – Antonio DJC
                                              Jun 11 '17 at 3:33










                                            • $begingroup$
                                              One thing is certain: my rhyming is rarely inadverten t.
                                              $endgroup$
                                              – marty cohen
                                              Jun 11 '17 at 4:49














                                            • 1




                                              $begingroup$
                                              Do a Google search for "proof square root of 2 is irrational". One link, which has a number of proofs, is here: en.wikipedia.org/wiki/Square_root_of_2
                                              $endgroup$
                                              – marty cohen
                                              May 22 '15 at 14:56










                                            • $begingroup$
                                              I found your this post. Thank you.
                                              $endgroup$
                                              – Bumblebee
                                              Jun 22 '15 at 7:43






                                            • 1




                                              $begingroup$
                                              Another link with may proofs is cut-the-knot.org/proofs/sq_root.shtml
                                              $endgroup$
                                              – Gerry Myerson
                                              Jul 21 '15 at 6:56






                                            • 1




                                              $begingroup$
                                              When writing your answer, did you mean to rhyme each time [: )]?
                                              $endgroup$
                                              – Antonio DJC
                                              Jun 11 '17 at 3:33










                                            • $begingroup$
                                              One thing is certain: my rhyming is rarely inadverten t.
                                              $endgroup$
                                              – marty cohen
                                              Jun 11 '17 at 4:49








                                            1




                                            1




                                            $begingroup$
                                            Do a Google search for "proof square root of 2 is irrational". One link, which has a number of proofs, is here: en.wikipedia.org/wiki/Square_root_of_2
                                            $endgroup$
                                            – marty cohen
                                            May 22 '15 at 14:56




                                            $begingroup$
                                            Do a Google search for "proof square root of 2 is irrational". One link, which has a number of proofs, is here: en.wikipedia.org/wiki/Square_root_of_2
                                            $endgroup$
                                            – marty cohen
                                            May 22 '15 at 14:56












                                            $begingroup$
                                            I found your this post. Thank you.
                                            $endgroup$
                                            – Bumblebee
                                            Jun 22 '15 at 7:43




                                            $begingroup$
                                            I found your this post. Thank you.
                                            $endgroup$
                                            – Bumblebee
                                            Jun 22 '15 at 7:43




                                            1




                                            1




                                            $begingroup$
                                            Another link with may proofs is cut-the-knot.org/proofs/sq_root.shtml
                                            $endgroup$
                                            – Gerry Myerson
                                            Jul 21 '15 at 6:56




                                            $begingroup$
                                            Another link with may proofs is cut-the-knot.org/proofs/sq_root.shtml
                                            $endgroup$
                                            – Gerry Myerson
                                            Jul 21 '15 at 6:56




                                            1




                                            1




                                            $begingroup$
                                            When writing your answer, did you mean to rhyme each time [: )]?
                                            $endgroup$
                                            – Antonio DJC
                                            Jun 11 '17 at 3:33




                                            $begingroup$
                                            When writing your answer, did you mean to rhyme each time [: )]?
                                            $endgroup$
                                            – Antonio DJC
                                            Jun 11 '17 at 3:33












                                            $begingroup$
                                            One thing is certain: my rhyming is rarely inadverten t.
                                            $endgroup$
                                            – marty cohen
                                            Jun 11 '17 at 4:49




                                            $begingroup$
                                            One thing is certain: my rhyming is rarely inadverten t.
                                            $endgroup$
                                            – marty cohen
                                            Jun 11 '17 at 4:49











                                            4












                                            $begingroup$

                                            Robin Chapman gives 14 proofs of $$sum_{n=1}^{infty}{1over n^2}={pi^2over6}$$ here.






                                            share|cite|improve this answer









                                            $endgroup$













                                            • $begingroup$
                                              This article provides a beautiful non-proof (Euler's proof) of this identity. Thank you.
                                              $endgroup$
                                              – Bumblebee
                                              Jul 17 '15 at 8:50


















                                            4












                                            $begingroup$

                                            Robin Chapman gives 14 proofs of $$sum_{n=1}^{infty}{1over n^2}={pi^2over6}$$ here.






                                            share|cite|improve this answer









                                            $endgroup$













                                            • $begingroup$
                                              This article provides a beautiful non-proof (Euler's proof) of this identity. Thank you.
                                              $endgroup$
                                              – Bumblebee
                                              Jul 17 '15 at 8:50
















                                            4












                                            4








                                            4





                                            $begingroup$

                                            Robin Chapman gives 14 proofs of $$sum_{n=1}^{infty}{1over n^2}={pi^2over6}$$ here.






                                            share|cite|improve this answer









                                            $endgroup$



                                            Robin Chapman gives 14 proofs of $$sum_{n=1}^{infty}{1over n^2}={pi^2over6}$$ here.







                                            share|cite|improve this answer












                                            share|cite|improve this answer



                                            share|cite|improve this answer










                                            answered Jul 15 '15 at 7:22









                                            Gerry MyersonGerry Myerson

                                            148k8152306




                                            148k8152306












                                            • $begingroup$
                                              This article provides a beautiful non-proof (Euler's proof) of this identity. Thank you.
                                              $endgroup$
                                              – Bumblebee
                                              Jul 17 '15 at 8:50




















                                            • $begingroup$
                                              This article provides a beautiful non-proof (Euler's proof) of this identity. Thank you.
                                              $endgroup$
                                              – Bumblebee
                                              Jul 17 '15 at 8:50


















                                            $begingroup$
                                            This article provides a beautiful non-proof (Euler's proof) of this identity. Thank you.
                                            $endgroup$
                                            – Bumblebee
                                            Jul 17 '15 at 8:50






                                            $begingroup$
                                            This article provides a beautiful non-proof (Euler's proof) of this identity. Thank you.
                                            $endgroup$
                                            – Bumblebee
                                            Jul 17 '15 at 8:50













                                            3












                                            $begingroup$

                                            For $ngeq kinmathbb{N}$, you can prove that $dfrac{n!}{k!cdot(n-k)!}$ is an integer from a combinatorial/counting argument. Establish that the formula gives the number of ways to choose a subset of $k$ from a set of $n$, and that automatically makes it an integer.



                                            Or you can prove that $dfrac{n!}{k!cdot(n-k)!}$ is an integer by showing that the highest power of any prime $p$ dividing the denominator is less than the highest power of $p$ dividing the numerator.



                                            Several proofs here.






                                            share|cite|improve this answer











                                            $endgroup$









                                            • 3




                                              $begingroup$
                                              You can also prove this using group actions: $S_n$ acts on the set ${1, 2, dotsc, n}$, and the stabilizer of any $k$-set from that set is $S_ktimes S_{n-k}$. So by the orbit-stabilizer theorem, the number of such $k$-sets is the number of orbits, which is $frac{|S_n|}{|S_k||S_{n-k}|} = frac{n!}{k!(n-k)!}$.
                                              $endgroup$
                                              – rogerl
                                              Jul 27 '15 at 17:27
















                                            3












                                            $begingroup$

                                            For $ngeq kinmathbb{N}$, you can prove that $dfrac{n!}{k!cdot(n-k)!}$ is an integer from a combinatorial/counting argument. Establish that the formula gives the number of ways to choose a subset of $k$ from a set of $n$, and that automatically makes it an integer.



                                            Or you can prove that $dfrac{n!}{k!cdot(n-k)!}$ is an integer by showing that the highest power of any prime $p$ dividing the denominator is less than the highest power of $p$ dividing the numerator.



                                            Several proofs here.






                                            share|cite|improve this answer











                                            $endgroup$









                                            • 3




                                              $begingroup$
                                              You can also prove this using group actions: $S_n$ acts on the set ${1, 2, dotsc, n}$, and the stabilizer of any $k$-set from that set is $S_ktimes S_{n-k}$. So by the orbit-stabilizer theorem, the number of such $k$-sets is the number of orbits, which is $frac{|S_n|}{|S_k||S_{n-k}|} = frac{n!}{k!(n-k)!}$.
                                              $endgroup$
                                              – rogerl
                                              Jul 27 '15 at 17:27














                                            3












                                            3








                                            3





                                            $begingroup$

                                            For $ngeq kinmathbb{N}$, you can prove that $dfrac{n!}{k!cdot(n-k)!}$ is an integer from a combinatorial/counting argument. Establish that the formula gives the number of ways to choose a subset of $k$ from a set of $n$, and that automatically makes it an integer.



                                            Or you can prove that $dfrac{n!}{k!cdot(n-k)!}$ is an integer by showing that the highest power of any prime $p$ dividing the denominator is less than the highest power of $p$ dividing the numerator.



                                            Several proofs here.






                                            share|cite|improve this answer











                                            $endgroup$



                                            For $ngeq kinmathbb{N}$, you can prove that $dfrac{n!}{k!cdot(n-k)!}$ is an integer from a combinatorial/counting argument. Establish that the formula gives the number of ways to choose a subset of $k$ from a set of $n$, and that automatically makes it an integer.



                                            Or you can prove that $dfrac{n!}{k!cdot(n-k)!}$ is an integer by showing that the highest power of any prime $p$ dividing the denominator is less than the highest power of $p$ dividing the numerator.



                                            Several proofs here.







                                            share|cite|improve this answer














                                            share|cite|improve this answer



                                            share|cite|improve this answer








                                            edited Apr 13 '17 at 12:20









                                            Community

                                            1




                                            1










                                            answered Jul 27 '15 at 16:40









                                            alex.jordanalex.jordan

                                            39.7k660122




                                            39.7k660122








                                            • 3




                                              $begingroup$
                                              You can also prove this using group actions: $S_n$ acts on the set ${1, 2, dotsc, n}$, and the stabilizer of any $k$-set from that set is $S_ktimes S_{n-k}$. So by the orbit-stabilizer theorem, the number of such $k$-sets is the number of orbits, which is $frac{|S_n|}{|S_k||S_{n-k}|} = frac{n!}{k!(n-k)!}$.
                                              $endgroup$
                                              – rogerl
                                              Jul 27 '15 at 17:27














                                            • 3




                                              $begingroup$
                                              You can also prove this using group actions: $S_n$ acts on the set ${1, 2, dotsc, n}$, and the stabilizer of any $k$-set from that set is $S_ktimes S_{n-k}$. So by the orbit-stabilizer theorem, the number of such $k$-sets is the number of orbits, which is $frac{|S_n|}{|S_k||S_{n-k}|} = frac{n!}{k!(n-k)!}$.
                                              $endgroup$
                                              – rogerl
                                              Jul 27 '15 at 17:27








                                            3




                                            3




                                            $begingroup$
                                            You can also prove this using group actions: $S_n$ acts on the set ${1, 2, dotsc, n}$, and the stabilizer of any $k$-set from that set is $S_ktimes S_{n-k}$. So by the orbit-stabilizer theorem, the number of such $k$-sets is the number of orbits, which is $frac{|S_n|}{|S_k||S_{n-k}|} = frac{n!}{k!(n-k)!}$.
                                            $endgroup$
                                            – rogerl
                                            Jul 27 '15 at 17:27




                                            $begingroup$
                                            You can also prove this using group actions: $S_n$ acts on the set ${1, 2, dotsc, n}$, and the stabilizer of any $k$-set from that set is $S_ktimes S_{n-k}$. So by the orbit-stabilizer theorem, the number of such $k$-sets is the number of orbits, which is $frac{|S_n|}{|S_k||S_{n-k}|} = frac{n!}{k!(n-k)!}$.
                                            $endgroup$
                                            – rogerl
                                            Jul 27 '15 at 17:27











                                            1












                                            $begingroup$

                                            Other examples of such many-proof theorems include:



                                            1) Harmonic series diverges



                                            There are at least 8 proofs of this fact based on different ideas.
                                            The first four of them can be found under this link: https://proofwiki.org/wiki/Harmonic_Series_is_Divergent



                                            The other four are of the following form:
                                            "If a subseries diverges, then the whole series diverges too. Now let's prove that the subseries of harmonic series, that formed only by reciprocals of primes diverges..."



                                            And there are at least four different ways to prove it: https://en.wikipedia.org/wiki/Divergence_of_the_sum_of_the_reciprocals_of_the_primes



                                            2) Variance of a random variable with support in $[0; 1]$ does not exceed $frac{1}{4}$



                                            There are at least three proofs of this, based on different ideas: What is the largest possible variance of a random variable on $[0; 1]$?



                                            3) No group is both a direct product and a direct product of non-trivial groups



                                            There are at least three proofs of this, based on different ideas: Does there exist a group that is both a free product and a direct product of nontrivial groups?



                                            4) $(mathbb{Q}, +)$ is not a direct product of any two non-trivial groups



                                            There are at least three proofs of this, based on different ideas: Is $(mathbb{Q},+)$ the direct product of two non-trivial subgroups?






                                            share|cite|improve this answer











                                            $endgroup$


















                                              1












                                              $begingroup$

                                              Other examples of such many-proof theorems include:



                                              1) Harmonic series diverges



                                              There are at least 8 proofs of this fact based on different ideas.
                                              The first four of them can be found under this link: https://proofwiki.org/wiki/Harmonic_Series_is_Divergent



                                              The other four are of the following form:
                                              "If a subseries diverges, then the whole series diverges too. Now let's prove that the subseries of harmonic series, that formed only by reciprocals of primes diverges..."



                                              And there are at least four different ways to prove it: https://en.wikipedia.org/wiki/Divergence_of_the_sum_of_the_reciprocals_of_the_primes



                                              2) Variance of a random variable with support in $[0; 1]$ does not exceed $frac{1}{4}$



                                              There are at least three proofs of this, based on different ideas: What is the largest possible variance of a random variable on $[0; 1]$?



                                              3) No group is both a direct product and a direct product of non-trivial groups



                                              There are at least three proofs of this, based on different ideas: Does there exist a group that is both a free product and a direct product of nontrivial groups?



                                              4) $(mathbb{Q}, +)$ is not a direct product of any two non-trivial groups



                                              There are at least three proofs of this, based on different ideas: Is $(mathbb{Q},+)$ the direct product of two non-trivial subgroups?






                                              share|cite|improve this answer











                                              $endgroup$
















                                                1












                                                1








                                                1





                                                $begingroup$

                                                Other examples of such many-proof theorems include:



                                                1) Harmonic series diverges



                                                There are at least 8 proofs of this fact based on different ideas.
                                                The first four of them can be found under this link: https://proofwiki.org/wiki/Harmonic_Series_is_Divergent



                                                The other four are of the following form:
                                                "If a subseries diverges, then the whole series diverges too. Now let's prove that the subseries of harmonic series, that formed only by reciprocals of primes diverges..."



                                                And there are at least four different ways to prove it: https://en.wikipedia.org/wiki/Divergence_of_the_sum_of_the_reciprocals_of_the_primes



                                                2) Variance of a random variable with support in $[0; 1]$ does not exceed $frac{1}{4}$



                                                There are at least three proofs of this, based on different ideas: What is the largest possible variance of a random variable on $[0; 1]$?



                                                3) No group is both a direct product and a direct product of non-trivial groups



                                                There are at least three proofs of this, based on different ideas: Does there exist a group that is both a free product and a direct product of nontrivial groups?



                                                4) $(mathbb{Q}, +)$ is not a direct product of any two non-trivial groups



                                                There are at least three proofs of this, based on different ideas: Is $(mathbb{Q},+)$ the direct product of two non-trivial subgroups?






                                                share|cite|improve this answer











                                                $endgroup$



                                                Other examples of such many-proof theorems include:



                                                1) Harmonic series diverges



                                                There are at least 8 proofs of this fact based on different ideas.
                                                The first four of them can be found under this link: https://proofwiki.org/wiki/Harmonic_Series_is_Divergent



                                                The other four are of the following form:
                                                "If a subseries diverges, then the whole series diverges too. Now let's prove that the subseries of harmonic series, that formed only by reciprocals of primes diverges..."



                                                And there are at least four different ways to prove it: https://en.wikipedia.org/wiki/Divergence_of_the_sum_of_the_reciprocals_of_the_primes



                                                2) Variance of a random variable with support in $[0; 1]$ does not exceed $frac{1}{4}$



                                                There are at least three proofs of this, based on different ideas: What is the largest possible variance of a random variable on $[0; 1]$?



                                                3) No group is both a direct product and a direct product of non-trivial groups



                                                There are at least three proofs of this, based on different ideas: Does there exist a group that is both a free product and a direct product of nontrivial groups?



                                                4) $(mathbb{Q}, +)$ is not a direct product of any two non-trivial groups



                                                There are at least three proofs of this, based on different ideas: Is $(mathbb{Q},+)$ the direct product of two non-trivial subgroups?







                                                share|cite|improve this answer














                                                share|cite|improve this answer



                                                share|cite|improve this answer








                                                edited Mar 24 at 11:36

























                                                answered Feb 7 at 19:32









                                                Yanior WegYanior Weg

                                                2,96321549




                                                2,96321549






























                                                    draft saved

                                                    draft discarded




















































                                                    Thanks for contributing an answer to Mathematics Stack Exchange!


                                                    • Please be sure to answer the question. Provide details and share your research!

                                                    But avoid



                                                    • Asking for help, clarification, or responding to other answers.

                                                    • Making statements based on opinion; back them up with references or personal experience.


                                                    Use MathJax to format equations. MathJax reference.


                                                    To learn more, see our tips on writing great answers.




                                                    draft saved


                                                    draft discarded














                                                    StackExchange.ready(
                                                    function () {
                                                    StackExchange.openid.initPostLogin('.new-post-login', 'https%3a%2f%2fmath.stackexchange.com%2fquestions%2f1009922%2fwhat-are-the-theorems-in-mathematics-which-can-be-proved-using-completely-differ%23new-answer', 'question_page');
                                                    }
                                                    );

                                                    Post as a guest















                                                    Required, but never shown





















































                                                    Required, but never shown














                                                    Required, but never shown












                                                    Required, but never shown







                                                    Required, but never shown

































                                                    Required, but never shown














                                                    Required, but never shown












                                                    Required, but never shown







                                                    Required, but never shown







                                                    Popular posts from this blog

                                                    Magento 2 - Add success message with knockout Planned maintenance scheduled April 23, 2019 at 23:30 UTC (7:30pm US/Eastern) Announcing the arrival of Valued Associate #679: Cesar Manara Unicorn Meta Zoo #1: Why another podcast?Success / Error message on ajax request$.widget is not a function when loading a homepage after add custom jQuery on custom themeHow can bind jQuery to current document in Magento 2 When template load by ajaxRedirect page using plugin in Magento 2Magento 2 - Update quantity and totals of cart page without page reload?Magento 2: Quote data not loaded on knockout checkoutMagento 2 : I need to change add to cart success message after adding product into cart through pluginMagento 2.2.5 How to add additional products to cart from new checkout step?Magento 2 Add error/success message with knockoutCan't validate Post Code on checkout page

                                                    Fil:Tokke komm.svg

                                                    Where did Arya get these scars? Unicorn Meta Zoo #1: Why another podcast? Announcing the arrival of Valued Associate #679: Cesar Manara Favourite questions and answers from the 1st quarter of 2019Why did Arya refuse to end it?Has the pronunciation of Arya Stark's name changed?Has Arya forgiven people?Why did Arya Stark lose her vision?Why can Arya still use the faces?Has the Narrow Sea become narrower?Does Arya Stark know how to make poisons outside of the House of Black and White?Why did Nymeria leave Arya?Why did Arya not kill the Lannister soldiers she encountered in the Riverlands?What is the current canonical age of Sansa, Bran and Arya Stark?